Tải bản đầy đủ (.doc) (15 trang)

CHUYÊN đề số học bài tập CỦNG cố một số nội DUNG cơ bản của lý THUYẾT ĐỒNG dư

Bạn đang xem bản rút gọn của tài liệu. Xem và tải ngay bản đầy đủ của tài liệu tại đây (327.19 KB, 15 trang )

CHUYÊN ĐỀ SỐ HỌC
BÀI TẬP CỦNG CỐ MỘT SỐ NỘI DUNG CƠ BẢN CỦA
LÝ THUYẾT ĐỒNG DƯ
I. Bài tập sử dụng Định lý Thặng dư Trung Hoa
Bài 1 Cho A là một tập con khác rỗng của Z +. Chứng minh rằng tồn tại số nguyên dương
n sao cho tập hợp nA = { nx / x ∈ A} chứa toàn lũy thừa của một số tự nhiên với số mũ
lớn hơn 1.
 Lời giải: Đặt A = {a1, a2,...., ak} và p1, p2,..., pk là k số nguyên tố phân biệt. Theo
định lý Thặng dư Trung Hoa, với mọi i = 1,2,..,k tồn tại số nguyên dương mi thỏa mãn
mi ≡ -1 ( mod pi) ; mi ≡ 0 ( mod pj) ( i ≠ j, j = 1,2,....,k)
Khi đó:
m1 + 1  p1

m2  p1

.......

mk  p1

m1  p2

m2 + 1  p2

.......

mk  p2

m2  pk

.......


mk + 1  pk

.....
m1  pk
Đến đây, ta đặt:

n = a1m1 . a2 m2 . a3m3 ...... a k mk

Ta có:
mk
m1 +1
m2
na1 = a1m1 +1 . a2 m2 . a3 m3 ...... a k mk = ( a1 p a p ..... a p ) p1
2 1
k 1
1
mk
m1
m2 +1
na2 = a1m1 . a2 m2 +1 . a3 m3 ...... a k mk = ( a p a p ..... a p ) p 2
1 2 2 2
k 2

......
mk +1
m1
m2
nak = a1m1 . a2 m2 . a3 m3 ...... a k mk +1 = ( a p a p ..... a p ) p k
1 k 2 k
k k


 ĐPCM.
Bài 2 Với mọi tập số nguyên {a1, a2, a3...., an } tồn tại số nguyên dương b để tập
{ ba1, ba2, ba3...., ban } bao gồm những lũy thừa đúng của một số nguyên.
 Lời giải:


Đặt p1, p2, p3,....., pn là các số nguyên tố có mặt trong phép phân tích a 1, a2,...., an
thành thừa số nguyên tố. Ta có: ai = p1bi1. p2bi2.....pkbik.
Ta biết rằng nếu một số nguyên được phân tích thành thừa số nguyên tố
p1c1 .... p m c m là một lũy thừa bậc q khi và chỉ khi c1, c2,...., cn chia hết cho q.

Đặt b = p1d1 . p2 d 2 . p3d 3 ..... p k d k . Ta sẽ chứng minh có thể tìm được d1, d2,...., dk
mà b thỏa mãn điều kiện đề bài.
Để có ba1 là lũy thừa bậc q1 của một số nguyên thì b11 + d1, b12 + d2,..., b1k + dk
chia hết cho q1. Tương tự với ba2, ba3,...., ban, ta có:
+ b21 + d1, b22 + d2,..., b2k + dk chia hết cho q2
.......
+ bn1 + d1, bn2 + d2,..., bnk + dk chia hết cho qn.
Từ đó phải xác định d1, d2,...., dk thoản mãn d1 ≡ - bi1 (mod qi); i = 1,2,..,n.
Tương tự với d2, d3,..., dk. Ta chọn q1, q2,...., qn là những số nguyên tố phân biệt.
Áp dụng định lý Thặng dư Trung Hoa, tồn tại d1, d2,..., dk. Từ đó tồn tại b (ĐPCM).
Bài 3 Chứng minh rằng với mọi số nguyên n, tồn tại một tập số nguyên n phần tử để
tổng các phần tử các tập con không rỗng của nó là một lũy thừa.
 Lời giải:
Ta xét tập số nguyên S = { x1, x2,...., xn }, tập S có 2n – 1 tập con không rỗng của S.
Đặt:

S1, S2, S3,...., S 2 n −1


Là tổng các phần tử của các tập này. Áp dụng bài tập 7, tồn tại số nguyên b thỏa mãn:
{ bS1, bS2, bS3,...., bS 2 n −1 }
Bao gồm toàn các lũy thừa. Từ đó ta chọn tập: F = { bx 1, bx2, bx3,...., bxn } thì tập F
thỏa mãn điều kiện đề bài.
Bài 4 Cho m là một số nguyên dương. Tìm số lượng nghiệm của phương trình
x2 ≡ x ( mod m).
 Lời giải: Giải sử m = p1a1 . p 2 a2 . p3a3 ...... pk ak .


a
Ta có: x2 ≡ x ( mod m). Từ đó x2 ≡ x ( mod pi ai ) với i = 1,2,3,..,k. ( pi ai , p j j ) = 1

với i ≠ j, i,j ∈ {1,2,..,n} hay x(x-1) ≡ 0 ( mod pi ai ) với i ∈ {1,2,..,n}. Vậy phương trình
x(x-1) ≡ 0 ( mod pi ai ) có 2 nghiệm là x ≡ 0 ( mod pi ai ) và x ≡ 1 ( mod pi ai ).
Áp dụng định lý Thặng dư Trung Hoa, hệ x ≡ ri ( mod pi ai ), i ∈ {1,2,..,n} với
ri ∈ {0,1} chỉ có 1 nghiệm mod m nhưng lại có 2 k cách chọn hệ. Vì vậy, phương trình có
2k nghiệm.
Bài 5 Cho n là số nguyên dương lẻ và n > 3. Gọi k, t là các số nguyên dương nhỏ
nhất sao cho kn + 1 và tn đều là số chính phương. Chứng minh n là số nguyên tố khi và
chỉ khi:


min {k,t} >

n
4

Lời giải:

1. Điều kiện cần: Giả sử n là số nguyên tố, khi đó:

n|tn và tn là chính phương nên n2|tn ⇒ n|t.
t ≥ n>

Từ đó:

n
4

Hơn nữa, đặt: a2 = kn + 1 thì a2 ≡ 1 (mod n). Kết hợp n nguyên tố nên a ≡ ± 1(mod n)
Nhưng vì a > 1 nên a ≥ n – 1 ( với n – 1 là số nhỏ nhất đồng dư với 1 mod n). Dẫn đến:
kn + 1 ≥ (n – 1)2 ⇒ k ≥ n – 2 ⇒ k >

n
n
, vì n > 3 nên n – 2 >
4
3

2. Điều kiện đủ:
• Nếu n chỉ có một ước số nguyên tố duy nhất, đặt n = p α với p ≥ 3 do n lẻ. nếu α
chẵn, ta lấy t = 1 <

n
thì tn = p α là số chính phương, mâu thuẫn với giải thiết. Nếu α lẻ,
4

α
n
α ≥ 3, ta lấy: t = p < p =
thì tn = p α +1 , α + 1 chẵn nên tn là số chính phương với

4
4

t<

n
, mâu thuẫn. Vậy α = 1 hay n là số nguyên tố.
4


• Nếu n có ít nhất 2 ước nguyên tố phân biệt. Khi đó ta có thể viết n dưới dạng
n = p α .m, trong đó p là một số nguyên tố lẻ, m là số nguyên dương lẻ, (m,p) = 1. Theo
định lý Thặng dư Trung Hoa, tồn tại số nguyên s sao cho:
s ≡ 1 ( mod p α )
s ≡ -1 (mod m)
n
2

Từ đó: n|s2 – 1. Hơn nữa ta có thể chọn s sao cho: s ≤ .
Vì s không đồng dư với 1 mod m nên s ≠ 1 và s không đồng dư với -1 mod p α nên
2
s ≠ -1. Dẫn đến s2 ≠ -1, từ đây lấy: k = s −1 thì k là số nguyên dương, hơn nữa
n

kn + 1 = s2 là số chính phương và k =

s 2 −1
n

Mâu thuẫn với điều kiện: min {k,t} >


n2
n
<
=
.
4
4
n ≤ n
s2

n
.
4

Vậy trường hợp này không thể xảy ra.
Vậy n là số nguyên tố.

II. Bài tập Phương trình đồng dư
Bài 6 : Cho p là số nguyên tố lẻ và k | p – 1. Khi đó phương trình x k – 1 = 0 (mod p) có
đúng k nghiệm phân biệt.


Lời giải:

Vì k | p – 1 nên dễ thấy xp-1 – 1 = (xk – 1)Q(x). Trong đó Q(x) là đa thức hệ số nguyên với
deg Q = p – 1 – k.
Gọi A ⊂ S = { 1, 2,..., p − 1} ; B ⊂ S tương ứng là nghiệm của 2 phương trình :
x p−1 ≡ 1 (mod p ) và Q( x) ≡ 0 (mod p )


Thì ta có A ∪ B = S ⇒| A | + | B |≥ p − 1 mà | A |≤ k ; | B |≤ p − 1 − k nên | A | + | B |= k .
Vậy xk – 1 = 0 (mod p) có đúng k nghiệm phân biệt.
k
k
Bài 7 : Chứng minh rằng nếu p là 1 số nguyên tố thì C p−1 ≡ (−1) (mod p) ∀1 ≤ k ≤ p − 1 .




Lời giải:

Xét đa thức f ( x) = ( x + 1) p−1 −

x p −1
.
x −1

Dễ thấy bậc của f (x) không quá p − 2 .

(1)

Mặt khác theo định lí Fermat nhỏ thì:

( x + 1) p−1 ≡ 1 (mod p)
x p ≡ x (mod p ) ⇒

x p − x + x +1
≡ 1 (mod p)
x +1


Do đó phương trình f ( x) ≡ 0 (mod p) có p − 1 nghiệm mod p là 0, 1, 2, …, p – 2.

(2)

Từ (1) và (2) suy ra mọi hệ số của f (x) đều chia hết cho p.
k
k
Vậy C p−1 ≡ (−1) (mod p) ∀1 ≤ k ≤ p − 1 .

Bài 8 : Chứng minh rằng nếu p là 1 số nguyên tố thì :
C pk Mp, ∀k = 1, p − 1



Lời giải:
p −1

(

)

Đặt f ( x) = ∑ C pk x k = ( x + 1) − x p + 1 .
k =1

p

Theo định lý Fermat nhỏ : ( x + 1) ≡ x + 1 ≡ x p + 1(mo d p), ∀x ∈ ¢.
p

Do đó phương trình f ( x) ≡ 0 (mo d p) có p nghiệm. Mà deg f = p − 1 nên mọi hệ số của

f ( x) đều chia hết cho p (đpcm).

Bài 9: Cho n là số tự nhiên, p là số nguyên tố, n ≥ p. Chứng minh rằng
n
Cnp ≡   (mod p) ,trong đó [ x ]
 p



là kí hiệu phần nguyên của x.

Lời giải:

Xét dãy n, n − 1,..., n − p + 1 . Dãy gồm p số tự nhiên liên tiếp nên có đúng một số của dãy
chia hết cho p, giả sử số đó là N.Ta có
N n
=
.
p  p 


Loại số N ra khỏi dãy và xét đồng dư modulo p, ta được dãy 1, 2,..., ( p − 1) (mod p) (xếp
theo thứ tự nào đó). Gọi Q là tích các số của dãy ( sau khi loại bỏ N). Ta có
Q=

n(n − 1)...(n − p + 1)
≡ ( p − 1)! (mod p).
N

Suy ra

NQ ≡ ( p − 1)! N (mod pN)

QN
N
≡ ( p − 1)! (mod p)
p
p

(1)

Vì (p, ( p − 1)!) = 1 nên ta được
NQ N

(mod p)
p!
p

(2)

n

p
Vậy Cn ≡   (mod p).
 p

Bài 10 : Xét các số tự nhiên m, n, k thoả mãn m n n m , n k k n . Chứng minh rằng m k k m .


Lời giải:


Từ giả thiết suy ra m n = an m , n k = bk n .
Ta có (m k ) n = ( m n ) = ( an m ) = a k ( n k ) = a k b m ( k m )
k

k

m

n

Suy ra ( m k ) ( k m ) .
n

n

Vậy m k k m .
Bài 11: cho p là số nguyên tố lẻ. chứng minh rằng với mọi n
-1 0 (mod


1 phương trình

)(*) có đúng p-1 nghiệm phân biệt.

Lời giải:

Chứng minh bằng quy nạp theo n. với n=1 khẳng định đúng. Giả sử khẳng định đúng với
n. xét phương trình
-1
Giả sử {

Ta có:

(mod

) (1)

} là p-1 nghiệm của (1).
-1 =

. Với mỗi i tồn tại duy nhất

{1,2,…,p}


Thỏa mãn: (p-1)

(mod p) vì (p-1)

không chia hết cho p

Đặt
Ta có:

-1 +(p-1)

(mod

)

Ta có :

Do vậy

là nghiệm của (*).

Đảo lại: Nếu

=>

(mod

) => i=j.

Vậy phương trình có ít nhất 9 nghiệm.
Giả sừ b là nghiệm bất kì của (*). Khi đó b cũng là nghiệm của (1). Vậy tồn tại
b=

. Ta có

 (p-1) t
Vậy t
 b

1=
chia hết cho p

(mod p) => t =
(mod

, t để


+pl => b=

)

vậy phương trình (*) có đúng p-1 nghiệm là

.

III. Hệ thặng dư
Bài 11: Cho a, b là các số nguyên Dương nguyên tố cùng nhau Số nguyên dương n được
gọi là số đẹp nếu tồn tại x,y sao cho
nếu không tồn tại

nguyên dương sao cho

CMR 1.

là số xấu lớn nhất.

. Số nguyên dương
.

được gọi là số xấu


2.CMR nếu

thì

là số đẹp khi và chỉ khi khi


là số xấu.
 Lời giải:
Ta chứng minh
Giả sử

là số xấu lớn nhất.
có nghiệm nguyên dương ⇒

là số đẹp, hay phương trình

(do



(

Khi đó phương trình (1) tương đương với
Vậy điều gỉa sử sai, tức
Ta chứng minh mọi
Do

vô lí.

là số xấu.
thì phương trình

nên

có nghiệm nguyên dương.


là HĐĐ mod

suy ra tồn tại

sao cho

.
Do

là số nguyên dương
Vậy tồn tại
Do đó,

nguyên dương sao cho

đều là số đẹp.

là số xấu lớn nhất.

2, Giả sử là số đẹp, khi đó tồn tại

sao cho

.

Khi đó
Giả sử

là số đẹp, khi đó tồn tại


nguyên dương sao cho


Từ (2) và (3) ta có
dương nên



nguyên

là số đẹp, mâu thuẫn với câu 1).

Vậy

là số xấu.

Ngược lại, đặt
Suy

. Do

ra

tồn

nên

là HĐĐ


tại

sao

.
cho

Theo giả thiết k
là số xấu, suy ra

Do

. Khi đó ta có



nguyên dương nên n là số đẹp

Bài 12: Cho số nguyên dương n và số nguyên tố p lớn hơn n+1. Chứng minh rằng đa
thức

không có nghiệm nguyên.
 Lời giải:
Ta có



trong đó các hệ số

dạng


gồm

Từ

suy ra tập

nhất

thừa

là HĐĐ mod

sao cho

số,

nên tồn tại duy

, hơn nữa

suy ra

không chia hết cho
Tương ứng với số
còn lại
Giả

trong đó có hệ số


không chia hết cho

đồng thời

hệ số

đều chia hết cho nhưng không chia hết cho
sử

PT



nghiệm

nguyên

Khi

đó


Theo
Suy

thì
ra




không


với

chia

hết



Vậy điều giả sử là sai, tức là PT

cho
nên

không có nghiệm nguyên, hay

nên
, mâu thuẫn
không có

nghiệm nguyên. (đpcm)
Bài 13: Cho dãy số

được xác định bởi

minh rằng dãy số

với


Chứng

chứa vô hạn số nguyên chia hết cho 7.

 Lời giải:
Giả sử chỉ có hữu hạn số trong dãy

chia hết cho 7 và

là số cuối cùng của

dãy chia hết cho 7. Từ công thức xác định của dãy ta có
Do

nên

Mặt khác

Do

nên tập

số
hơn

là HĐĐ mod 7, suy ra trong bảy
tồn tại một số chia hết cho 7 mà số này lại lớn

. Điều này mâu thuẫn với giả sử


sử sai nên ta có đpcm.

là số cuối cùng trong dãy chia hết cho 7. Do giả


Bài 14: Cho các số nguyên không âm
tại bốn số nguyên phân biệt

Chứng minh rằng tồn
thỏa mãn

 Lời giải:
Xét các tổng

có tất cả

Giả sử không tồn tại bốn số

tổng như vậy.

với

sao cho

thì

là HĐĐ mod 5050. Từ đó

là một số lẻ (1)

Mặt khác

là một số chẵn (2)

Ta thấy (1) và (2) mâu thuẫn. Vậy tồn tại hai tổng
sao cho
bốn số phân biệt và bốn số



với

lúc đó
thỏa mãn

phải là
.

Bài 15: Cho m là một số nguyên dương. Tìm số nghiệm của phương trình:
x 2 ≡ x (mod m).

 Lời giải:
Giả sử m = p1α p2α ... pkα . Ta có: x 2 ≡ x (mod m)
1

2

k

α

⇔ x 2 ≡ x (mod pi i ) ∀i = 1, 2,..., k

Hay x( x − 1) ≡ 0 (mod piα ) ∀i = 1, 2,..., k
i

Vì ( x; x − 1) = 1 nên phương trình x( x − 1) ≡ 0 (mod piα ) có 2 nghiệm mod piα là: x ≡ 0
i

α
α
(mod pi ) hoặc x ≡ 1 (mod pi ).
i

i

Theo Định lý Thặng dư Trung hoa: Với mỗi bộ { r1 , r2 ,..., rk } thì phương trình:
x ≡ ri (mod piαi ) ∀i = 1, 2,..., k luôn có một nghiệm duy nhất mod m

i


α
α
Do mỗi phương trình x( x − 1) ≡ 0 (mod pi ) đều có hai nghiệm mod pi nên phương trình
i

i

đã cho có 2k nghiệm.


Bài 16: Chứng minh rằng : với mọi số nguyên dương n , tồn tại số tự nhiên n gồm n chữ
số đều lẻ và nó chia hết cho 5n.
 Lời giải:
Xét số Xn = a1 a2 a3… an =
dương

i

+

lẻ với mọi i=1,2,3,,,n và a nguyên

Ta sẽ chứng minh bài toán bằng quy nạp : với n=1 thì

a1 =5

Gỉa sử mệnh đề đứng với n . Cần chứng minh mệnh đề đúng với n+1
Xét 5 số sau đây:
a1 = 1a1 a2 a3… an =
a2=3a1 a2 a3… an =
a3= 5 a1 a2a3… an =
a4= 7a1 a2 a3… an =
a5= 9a1 a2 a3… an =
Do B=
là HĐĐ mod 5 nên : C=
cũng là HĐĐ mod 5 nên tồn tại một số trong hệ chia hết cho 5.
Trong 5 số a1,a2,a3,a4,a5 có duy nhất một số trong C chia hết cho 5(n+1) mà số này
gồm (n+1) chữ số lẻ .
Do đó mệnh đề đúng với n+1.điều giả sử đúng với mọi n


*

Vậy với mọi số nguyên dương n , tồn tại số tự nhiên n gồm n chữ số đều lẻ và nó chia hết
cho 5n.

IV. Định lý Fermat nhỏ
Bài 17 Tìm tất cả các số tự nhiên m sao cho nếu m n ≡ 1 ( mod n) với n là số nguyên
dương tùy ý thì m ≡ 1 ( mod n).


 Lời giải:
+) Giả sử m > 2. Khi đó ta có:
m( m −1) ≡ [ 1 + (m − 1)]
2

( m −1)2

2
≡ 1 ( mod (m − 1) )

Mặt khác m − 1 không chia hết cho (m − 1) 2 với m > 2 .
Do đó mọi giá trị m > 2 không thỏa mãn đề bài.
+) m = 1 thỏa mãn.
+) m = 2 ta có:
2n ≡ 1 ( mod n) ∀n > 1

Giả sử p là ước nguyên tố nhỏ nhất của n, khi đó:
2n ≡ 1 ( mod p)

=> p lẻ. Ta có (n; p − 1) = 1 . Khi đó ∃a, b ∈ Z sao cho

an + b( p − 1) = 1

Theo định lí Fermat nhỏ ta có:
21 = 2na.2b ( p−1) ≡ 1 (mod p) ( mâu thuẫn)

Mâu thuẫn này chứng tỏ rằng giả thiết m n ≡ 1 ( mod n) không bao giờ xảy ra.
Vậy ta có mệnh đề m n ≡ 1 ( mod n) => m ≡ 1 ( mod n) đúng khi và chỉ khi m=1 hoặc m=2
9 p −1
Bài 18 Cho p là số nguyên tố lẻ. Đặt m =
. CMR: m là một hợp số lẻ, không chia hết
8

cho 3 và 3m−1 ≡ 1 ( mod m)
 Lời giải:
Ta có:
m=

9 p −1 3p −1 3p +1
=
.
8
2
4

Dễ thấy

3p −1
3p +1

đều là số nguyên dương lớn hơn 1 nên m là hợp số.

2
4

Mặt khác
9 p −1
m=
= 9 p −1 + 9 p − 2 + ... + 9 + 1 => m ≡ 1 ( mod 3) hay m không chia hết cho 3.
8

Ta thấy 9k có tận cùng là 9 với k lẻ, có tận cùng là 1 nếu k chẵn
m = 9 p −1 + 9 p − 2 + ... + 9 + 1 = (9 p −1 + 9 p − 2 ) + (9 p −3 + 9 p − 4 ) + .... + (9 2 + 9) + 1

Do p lẻ => m có tận cùng bằng 1=> m lẻ.


Theo định lí Fermat nhỏ: 9 p−1 ≡ 1 ( mod p) => 9 p − 9Mp
mà 9 p − 9M8, (8; p) = 1
Do đó 9 p − 9M8 p hay m − 1 =

9p − 9
Mp
8

Do m lẻ => m-1 chẵn => m − 1M2 p => 3m −1 − 1M32 p − 1 => 3m −1 − 1Mm ( do 32 p − 1 = 8m )
hay 3m−1 ≡ 1 ( mod m)
x
x
y
y
p

p
q
q
Bài 19 Tìm các số nguyên tố p, q thỏa mãn ( 7 − 5 ) ( 7 − 5 ) ( 7 − 5 ) ( 7 − 5 )

chia hết cho pq.
 Lời giải:
Bổ

đề: (quen

x
x
Cho ( 7 − 5 ) ⋮p,

thuộc)

(7

y

− 5 y ) ⋮p với x nhỏ

nhất

thì y⋮x

Áp

dụng:


Ta

thấy p,q≠5,7

p
p
p −1
p −1
TH1: ( 7 − 5 ) ⋮p theo Fermat nhỏ thì ( 7 − 5 ) ⋮p

⇔ ( 7 p − 5 p−1.7 ) Mp
⇒ 2.5 p−1 Mp
⇒ 2Mp
⇒ p=2

(7

2

− 52 ) ( 7 q − 5 q )

2
q
⇒ 12 ( 7 − 5q ) Mq

Mq

⇒ 2.5 p−1 Mp ⇒ 2Mp ⇒ p = 2


⇒ 12.2.5 p−1 Mq
⇒ 12.2Mq ⇒ q ∈ { 2;3}

Khi ấy:

(7

2

− 5 2 ) ( 7 q − 5q )

2
q
⇒ 12 ( 7 − 5q ) Mq

Mq

⇒ 12.2.5 p−1 Mq
⇒ 12.2Mq ⇒ q ∈ { 2;3}

TH2:

(7

p

− 5p ) ,

p


⇒ 7 q − 5q ⋮p


q
q
Nếu ( 7 − 5 ) ⋮q thì

cm

tương

tự

q
q
Nếu ( 7 − 5 ) ,

p
p
như ( 7 − 5 ) ⋮p
p
p
q⇒ ( 7 − 5 ) ⋮q

p
p
q
q
vậy ( 7 − 5 ) ⋮q và ( 7 − 5 ) ⋮p


Như

k
k
Gọi ( 7 − 5 ) ⋮q với k đạt GTNN khi ấy theo bổ đề thì p⋮k nên mà p nguyên tố nên k=1,p

Khi k=1⇒2⋮q⇒q=2 và

dễ

tìm

ra p

p
p
Khi k=p⇒ ( 7 − 5 ) ⋮q với p là số nhỏ nhất thỏa đề, khi đó dẫn đến vô lý vì vẫn còn số nhỏ

hơn

thỏa

mãn

q −1
q −1
là ( 7 − 5 ) ⋮q (theo

Fermat


nhỏ)

nên





Do đó trường hợp này có nghiệm giống TH1
Vậy (p,q)=(2,3),(3,2),(2,2).
Bài 20 Giả sử m, p là các số nguyên tố khác nhau. CMR nếu với số tự nhiên nào đó, p là
ước của số x m−1 + x m −2 + ... + 1 thì ta có p ≡ 1 ( mod m)
 Lời giải:
Giả sử p = mk + r , 1 ≤ r ≤ m − 1 . Ta có:
x m −1 + x m− 2 + ... + 1 Mp => x m ≡ 1 ( mod p) (1) => x không chia hết cho p.

Ta chứng minh: x r −1 ≡ 1 ( mod p) (2).
+) Nếu p < m thì (2) đúng do Fermat
p −1
Giả sử p > m . Nâng cả 2 vế của (1) lên lũy thừa k =
ta được:
m

x p −r ≡ 1 ( mod p)

Theo Fermat nhỏ:
x p −1 ≡ 1 ( mod p)

=> x p − r ( x r −1 − 1) ≡ 0 ( mod p)
=> x r −1 ≡ 1 ( mod p)

Cần chứng minh r = 1 . Giả sử r > 1 . Khi đó (m; r − 1) = 1 . Ta có:
( x m −1 − 1; x r −1 − 1) = x ( x ;r −1) − 1 = x − 1

Từ (1) và (2)=> x m − 1Mp, x r −1 − 1Mp
=> x − 1Mp => x ≡ 1 ( mod p)
=> P( x) ≡ m ( mod p) ( trái với giả thiết P( x)Mp )


Do đó r = 1 => p ≡ 1 ( mod m). ( đpcm)



×